Question

4. -/10 points My Notes Ask Your Teacher The figure below shows a top view of a bar that can slide on two frictionless rails.
0 0
Add a comment Improve this question Transcribed image text
Answer #1

01 force = BIL = RBL I BLU F = B22²0 na = 2.5x1.22x1.5 6.4 F= 2.11NT E Power F.1=2.18 15= 3.164 I

Add a comment
Know the answer?
Add Answer to:
4. -/10 points My Notes Ask Your Teacher The figure below shows a top view of a bar that can slide on two frictionl...
Your Answer:

Post as a guest

Your Name:

What's your source?

Earn Coins

Coins can be redeemed for fabulous gifts.

Not the answer you're looking for? Ask your own homework help question. Our experts will answer your question WITHIN MINUTES for Free.
Similar Homework Help Questions
  • The figure below shows a top view of a bar that can slide on two frictionless...

    The figure below shows a top view of a bar that can slide on two frictionless rails. The resistor is R = 5.00 Ω, and a 2.50-T magnetic field is directed perpendicularly downward, into the page. Let ℓ = 1.20 m. A vertical bar and two parallel horizontal rails lie in the plane of the page, in a region of uniform magnetic field, vector Bin, pointing into the page. The parallel rails run from left to right, with one a...

  • The figure below shows a top view of a bar that can slide on two frictionless...

    The figure below shows a top view of a bar that can slide on two frictionless rails. The resistor is R - 6.80 O, and a 2.50-T magnetic field is directed perpendicularly downward, into the page. Let ! - 1.20 m. XX XX R (a) Calculate the applied force required to move the bar to the right at a constant speed of 1.90 m/s. N (to the right) (6) At what rate is energy delivered to the resistor? w

  • The figure below shows a top view of a bar that can slide on two frictionless...

    The figure below shows a top view of a bar that can slide on two frictionless rails. The resistor is R = 6.40 s, and a 2.50-T magnetic field is directed perpendicularly downward, into the page. Let l = 1.20 m. X X X X X X Bin X X X X X x X x X X X R X X X X X X *F app X X X X X X X X X (a) Calculate the...

  • The adjacent figure shows a top view of a bar that can slide on two frictionless...

    The adjacent figure shows a top view of a bar that can slide on two frictionless rails. The resistance is R = 6.2. A 2.5 T magnetic field is directed perpendicularly downward into the paper. Let l = 1.2 m. (a) Calculate the applied force required to move the bar to the right at a constant speed of 2 m/s. X xx xx x x Bin (b) At what rate is energy delivered to the resistor? app

  • please help me!! will comment and rate Thank you!! The figure below shows a top view...

    please help me!! will comment and rate Thank you!! The figure below shows a top view of a bar that can slide on two frictionless rails. The resistor is R 7.000, and a 2.50-T magnetic field is directed perpendicularly downward, into the page. Let l = 1.20 m. X X X X X X X X XXXIX X X X X RZ x xxxxx X X X X P X X X X X X X X (a) Calculate the...

  • QUESTION 12 2, and a 2.50-T magnetic field is directed perpendicularly downward, The figure below shows...

    QUESTION 12 2, and a 2.50-T magnetic field is directed perpendicularly downward, The figure below shows a top view of a bar that can slide on two frictionless rails. The resistor is R = 6.20 into the page. Let l = 1.20 m. x * X * * * BAM * * * * * * * * X X X * * * * * * X X X * * * * * R . * * X...

  • The figure below shows a bar of mass m = 0.280 kg that can slide without...

    The figure below shows a bar of mass m = 0.280 kg that can slide without friction on a pair of rails separated by a distance ℓ = 1.20 m and located on an inclined plane that makes an angle θ = 29.5° with respect to the ground. The resistance of the resistor is R = 2.20 Ω, and a uniform magnetic field of magnitude B = 0.500 T is directed downward, perpendicular to the ground, over the entire region...

  • My Notes Ask Your Teacher 3. -13 points SerPSE9 31.P.029. A conducting rod of length moves on two horizontal, frict...

    My Notes Ask Your Teacher 3. -13 points SerPSE9 31.P.029. A conducting rod of length moves on two horizontal, frictionless rails, as in the figure. A constant force of 1.00 N moves the bar at 1.00 m/s through a magnetic field B that is directed into the monitor (a) What is the current through the 7.00 resistor R? (b) What is the rate at which energy is delivered to the resistor? (c) What is the mechanical power delivered by the...

  • 13. (-/4 Points] DETAILS SERPSE10 30.A.P.035. MY NOTES ASK YOUR TEACHER PRACTICE ANOTHER A conducting rod...

    13. (-/4 Points] DETAILS SERPSE10 30.A.P.035. MY NOTES ASK YOUR TEACHER PRACTICE ANOTHER A conducting rod of length 1 - 35.0 cm is free to slide on two parallel conducting bars as shown in the figure below. Two resistors R. - 2.000 and R, - 5.000 are corected across the ends of the bars to form a loop. A constant magnetic field 8 - 2.95 T is directed perpendicularly into the page. An external agent pulls the rod to the...

  • Question 31.8 and question 31.27 000 W A strong a unifrm magnetic field of 1.60 T...

    Question 31.8 and question 31.27 000 W A strong a unifrm magnetic field of 1.60 T over a cross-sectional area of 0320 m2. A col' having 230 turns and a total resistance of 19.0 n is placed around the electromagnet. The current in the electromagnet is then smoothly reduced until it reaches zero in 20.0 ms. What is the current induced in the col? 31 P 027 MI F The figure below shows a top view of a bar that...

ADVERTISEMENT
Free Homework Help App
Download From Google Play
Scan Your Homework
to Get Instant Free Answers
Need Online Homework Help?
Ask a Question
Get Answers For Free
Most questions answered within 3 hours.
ADVERTISEMENT
ADVERTISEMENT
ADVERTISEMENT